Exam 1

  1. Question

    For the matrix A=(16416445691663328492858). \begin{aligned} A &= \left( \begin{array}{rrrr} 16 & 4 & 16 & -4 \\ 4 & 5 & 6 & -9 \\ 16 & 6 & 33 & -28 \\ -4 & -9 & -28 & 58 \end{array} \right). \end{aligned} compute the matrix L=(ij)1i,j4L = (\ell_{ij})_{1 \leq i,j \leq 4} from the Cholesky decomposition A=LLA = L L^\top.

    Which of the following statements are true?


    1. 41=1\ell_{41} = -1
    2. 44<4\ell_{44} < 4
    3. 22=1\ell_{22} = -1
    4. 11>0\ell_{11} > 0
    5. 321\ell_{32} \le 1

    Solution

    The decomposition yields L=(4000120041401454) \begin{aligned} L &= \left( \begin{array}{rrrr} 4 & 0 & 0 & 0 \\ 1 & 2 & 0 & 0 \\ 4 & 1 & 4 & 0 \\ -1 & -4 & -5 & 4 \end{array} \right) \end{aligned} and hence:


    1. True. 41=1\ell_{41} = -1
    2. False. 44=44\ell_{44} = 4 \nless 4
    3. False. 22=21\ell_{22} = 2 \neq -1
    4. True. 11=4\ell_{11} = 4
    5. True. 32=1\ell_{32} = 1